LSAT and Law School Admissions Forum

Get expert LSAT preparation and law school admissions advice from PowerScore Test Preparation.

User avatar
 Dave Killoran
PowerScore Staff
  • PowerScore Staff
  • Posts: 5852
  • Joined: Mar 25, 2011
|
#85107
Complete Question Explanation
(The complete setup for this game can be found here: lsat/viewtopic.php?f=351&t=3899)

The correct answer choice is (B)

If no green bulbs are selected, then each bulb is either purple or yellow:

G2-Q10-d1.png

Because the question asks for how many different color sequences are possible, the best approach is to test light 1, first as purple, and then as yellow.

When light 1 is purple:

When light 1 is purple, according to the first rule light 2 must be yellow. But, when light 2 is yellow, from the contrapositive of the third rule, light 3 cannot be purple or yellow, and must be green. But, as this violates the condition in this question stem, this does not allow for a workable solution. Thus, there are no viable solutions when light 1 is purple.
When light 1 is yellow:
When light 1 is yellow, then light 2 can be purple or yellow:
G2-Q10-d2.png
When light 2 is yellow, from the contrapositive of the third rule, light 3 cannot be purple or yellow, and must be green. But, as this violates the condition in this question stem, this does not allow for a workable solution. Thus, there are no possible solutions when light 1 is yellow, and light 2 is yellow.
When light 2 is purple, then light three can be purple or yellow:
G2-Q10-d3.png
When light 3 is purple or yellow, no violations occur, meaning that there are two possible solutions when light 1 is yellow and light 2 is purple. Thus, two is the correct answer, and answer choice (B) is correct.
 theTRSlegend
  • Posts: 3
  • Joined: May 08, 2021
|
#87360
I had a little trouble with this question but reviewed and think I understand:

If we cannot use G, then we only have P & Y.

Due to Rule 1, if P is first then Y is second. However, Due to Rule 3, we cannot put Y second because P must maintain that spot (assuming no G). Thus the only option would be to put Y first only, or Y first and third, leading to YPP or YPY as the only two possible unique combos. Is this line of thinking correct?

Thanks!
 Robert Carroll
PowerScore Staff
  • PowerScore Staff
  • Posts: 1787
  • Joined: Dec 06, 2013
|
#87370
TRS,

That's exactly the correct way of thinking. I got the same conclusion a little differently at the end, or maybe I just thought it using different words: 1 is definitely yellow, so 1 and 2 are completely fixed, and only 3 can be different (although not green), so two different total sequences. Thus, answer choice (B) is correct.

Robert Carroll

Get the most out of your LSAT Prep Plus subscription.

Analyze and track your performance with our Testing and Analytics Package.